Ito's Formula Question: Understanding g and h Functions in Ito's Formula

  • Thread starter Thread starter Gregg
  • Start date Start date
  • Tags Tags
    Formula
Click For Summary
The discussion centers on understanding the functions g and h in Ito's formula, particularly in the context of calculating the difference F(Y(t)) - F(Y(0)) for F(t, x) = sin(x). The main inquiry is whether the relationship Y(t) - Y(0) = ∫₀ᵗ dW(t) imposes additional conditions on g(t) and h(t), and if these functions represent the mean and variance of the stochastic process Y(t). Participants seek clarification on the significance of g and h, and whether their values can be derived from the provided information. The conversation emphasizes the need to express the right-hand side of the equation using Riemann and Ito integrals while exploring the implications of the stochastic nature of Y(t).
Gregg
Messages
452
Reaction score
0
I have a question about the functions g and h in the Ito formula (below). The question is about finding

##F(Y(t))-F(Y(0)) = \sin(Y(t)) - \sin(Y(0))##

given that

## Y(t) - Y(0) = \int_0^t dW(t)##

Ito's formula:

##F(b,Y(b)) - F(b,Y(b)) = \int_a^b \frac{\partial F}{\partial t}(t,Y(t)) dt + \int_a^b (g(t) \frac{\partial F}{\partial x}(t,Y(t)) + \frac{1}{2}h^2(t) \frac{\partial^2 F }{\partial x^2} (t,Y(t)) dt + \int_a^b h(t) \frac{\partial f}{\partial x} (t,Y(t)) dW(t) ##

For example, we have

##F(t,x) = \sin(x)## so

##\frac{\partial F}{\partial t} = \frac{\partial Y}{\partial t} (t) \cos (Y(t)) ## etc.

Plugging all the values in

##\sin(Y(t)) - \sin(Y(0))=\int_0^t \frac{\partial Y}{\partial t}(t) \cos(Y(t)) dt + \int_0^t g(t) \cos(Y(t)) - \frac{1}{2}h^2(t) \sin(Y(t)) dt + \int_0^t h(t) dW(t) ##

does the ## Y(t) - Y(0) = \int_0^t dW(t)## set any extra condition on the ##h(t)## and ##g(t)## in the above expression? Or is it fine left as is, what are the two functions h,g? Are they mean and variance? This would mean that the mean of ##Y_t## is 0 and the variance is ##1##? Does this mean that the RV a corresponding g and h? how do I work this out?
 
Physics news on Phys.org
Gregg said:
I have a question about the functions g and h in the Ito formula (below). The question is about finding

##F(Y(t))-F(Y(0)) = \sin(Y(t)) - \sin(Y(0))##

given that

## Y(t) - Y(0) = \int_0^t dW(t)##

Ito's formula:

##F(b,Y(b)) - F(b,Y(b)) = \int_a^b \frac{\partial F}{\partial t}(t,Y(t)) dt + \int_a^b (g(t) \frac{\partial F}{\partial x}(t,Y(t)) + \frac{1}{2}h^2(t) \frac{\partial^2 F }{\partial x^2} (t,Y(t)) dt + \int_a^b h(t) \frac{\partial f}{\partial x} (t,Y(t)) dW(t) ##

For example, we have

##F(t,x) = \sin(x)## so

##\frac{\partial F}{\partial t} = \frac{\partial Y}{\partial t} (t) \cos (Y(t)) ## etc.

Plugging all the values in

##\sin(Y(t)) - \sin(Y(0))=\int_0^t \frac{\partial Y}{\partial t}(t) \cos(Y(t)) dt + \int_0^t g(t) \cos(Y(t)) - \frac{1}{2}h^2(t) \sin(Y(t)) dt + \int_0^t h(t) dW(t) ##

does the ## Y(t) - Y(0) = \int_0^t dW(t)## set any extra condition on the ##h(t)## and ##g(t)## in the above expression? Or is it fine left as is, what are the two functions h,g? Are they mean and variance? This would mean that the mean of ##Y_t## is 0 and the variance is ##1##? Does this mean that the RV a corresponding g and h? how do I work this out?

What, exactly, do you mean by "finding" F(Y(t))? Do you want the expected value? The probability distribution? Something else?
 
##F(Y(t))-F(Y(0)) = \sin(Y(t)) - \sin(Y(0)) ## this is the relationship between the RHS and the Ito formula which is given in terms of ## F(b,Y(b) - F(a, T(a)) ## it should read


## F(t, Y(t))-F(0, Y(0)) = \sin(Y(t)) - \sin(Y(0)) ##

I am to use the Ito formula to get an expression for the RHS in terms of Riemann and Ito integrals. My question is, once we have that form, what is the significance of the functions g,h, can their values be computed from the given information?
 
Question: A clock's minute hand has length 4 and its hour hand has length 3. What is the distance between the tips at the moment when it is increasing most rapidly?(Putnam Exam Question) Answer: Making assumption that both the hands moves at constant angular velocities, the answer is ## \sqrt{7} .## But don't you think this assumption is somewhat doubtful and wrong?

Similar threads

  • · Replies 11 ·
Replies
11
Views
3K
Replies
2
Views
2K
  • · Replies 3 ·
Replies
3
Views
2K
Replies
12
Views
2K
  • · Replies 3 ·
Replies
3
Views
2K
Replies
10
Views
3K
  • · Replies 1 ·
Replies
1
Views
1K
Replies
8
Views
2K
Replies
1
Views
2K
Replies
3
Views
2K